LSAT 8 – Section 1 – Question 10

You need a full course to see this video. Enroll now and get started in less than a minute.

Target time: 1:42

This is question data from the 7Sage LSAT Scorer. You can score your LSATs, track your results, and analyze your performance with pretty charts and vital statistics - all with a Free Account ← sign up in less than 10 seconds

Question
QuickView
Type Tags Answer
Choices
Curve Question
Difficulty
Psg/Game/S
Difficulty
Explanation
PT8 S1 Q10
+LR
Strengthen +Streng
A
2%
160
B
47%
166
C
0%
151
D
1%
150
E
49%
161
152
165
178
+Hardest 148.775 +SubsectionMedium
This page shows a recording of a live class. We're working hard to create our standard, concise explanation videos for the questions in this PrepTest. Thank you for your patience!

This is a strengthening question, indicated by: Which one of the following, if true, most strengthens the argument?

Our argument claims that if a library shared by two towns was moved from a smaller building in the lower population town to a larger building in the higher population town, that the library would be within walking distance of the library. It is worth noting that we have a conditional conclusion; if we move the library, then it will have more users within walking distance. It is important to remember that the truth of a conditional statement is independent from the truth of either of its conditions. For example, the statement “if it rains then the sidewalk will be wet” could be true even if it never rained and the sidewalk was never wet; the truth of a conditional is about the relationship between the two conditions, more specifically, whether the sufficient term is enough for the necessary term to be true.

Now when judging an argument with a conditional conclusion, we can either keep this in mind, or kick up the sufficient condition by making it a premise. In this case, we would simply add that the public library will be relocated, and now the conclusion we want to strengthen is that there will be more users within walking distance. The problem we should recognize with this argument is that we have a term shift; our support is about the total population of the two towns, but our conclusion is about specifically library users and even more specifically those within walking distance. It could be true that there are more people living in central Glenwood than central Redville, and yet there could be less library users, or the city could be less dense so that there are actually less people in general within walking distance. An answer choice which filled this gap between our support and our conclusion would be a great strengthening answer. Let’s see what we get:

Answer Choice (A) For all we know the old area between the two towns had more people within walking distance! Just because it was between the two cities we can’t assume there weren’t lots of people living nearby.

Correct Answer Choice (B) This helps fills our gap; now that we know that the cities are of comparable size, if one has a larger total population then it will have a greater population density, which would support that more people would be in walking distance of the new location.

Answer Choice (C) Our conclusion is about the number of library users in walking distance; the size of the building itself has no obvious impact on that factor.

Answer Choice (D) We don’t care about these out-of-city people; our conclusion is about a difference in the number of walking distance library users, who would all be within the city.

Answer Choice (E) The key words here are currently and considered. We aren’t interested in what people consider walking distance, we want to know how many library users are in actual walking distance. If the people are currently walking to the library, then regardless of what people consider they are by definition within walking distance. So all this answer really tells us is that people do walk to the current library location; this doesn’t give us any info to support that the new location will be better.

Take PrepTest

Review Results

Leave a Reply